7
$\begingroup$

I'm trying to prove that if I have a sequence of continuously differentiable functions $f_n$ that converge uniformly on $[a,b]$, then $\{f_n\}$ is equicontinuous for all $x_0 \in [a, b]$.

My idea is to use uniform convergence to deal with the "tail" and then use continuity to deal with the finitely many $f_n$'s left. But I'm having trouble writing it down.

$\endgroup$

3 Answers 3

6
$\begingroup$

There is no need to assume differentiability. Let $\epsilon>0$ and $x_0$ be given. Uniform convergence implies that the sequence is uniformly Cauchy. Choose $n_0$ such that $|f_n(x)-f_{n_0}(x)|<\epsilon$ for all $x$ and all $n\ge n_0$. Choose $\delta_k \ (k\in \{1,\ldots,n_0\})$ such that $|x-x_0|<\delta_k$ implies $|f_k(x)-f_k(x_0)|<\epsilon$. Put $\delta=\min_k{\delta_k}$.

Now, if $n\ge n_0$ and $|x-x_0|<\delta$, the triangle inequality yields $|f_n(x)-f_n(x_0)|\le |f_n(x)-f_{n_0}(x)|+|f_{n_0}(x)-f_{n_0}(x_0)|+|f_{n_0}(x_0)-f_n(x_0)|<3\epsilon$.

If $k\le n_0$, then $|x-x_0|<\delta\le\delta_k$ implies $|f_k(x)-f_k(x_0)|<\epsilon$ by choice of $\delta_k$ above.

$\endgroup$
2
  • $\begingroup$ I have a question here: Is it necessary for the domain to be compact for the result to hold? I don't see any compactness being used here in the proof. $\endgroup$
    – Lucas
    Jan 21 at 8:49
  • $\begingroup$ @Lucas No I think this works for an arbitrary domain. $\endgroup$ Jan 21 at 16:32
2
$\begingroup$

If you have a sequence of continuous functions $f_n \to f$ uniformly, it suffices to assume they are continuous. The sequence, along with its limit, is compact in the uniform norm. By Arzela-Ascoli, it is equicontinous.

$\endgroup$
1
$\begingroup$

See this

$$ |f_n(x)-f_n(y)| \leq |f'_n(\eta)||x-y|,\quad \eta \in (x,y) $$

by mean value theorem which implies

$$|f_n(x)-f_n(y)| \leq M |x-y| $$

since $|f'_n(x)|$ is continuous on $[a,b]$. I think there is nothing left for you to do except some details.

$\endgroup$
2
  • $\begingroup$ Would you care explaining what's the downvote for? $\endgroup$
    – science
    Mar 29, 2015 at 19:53
  • 1
    $\begingroup$ You assumed that $f'_n$ is uniformly bounded. That might not be true even if $f_n$ converges uniformly to a function. $\endgroup$ Nov 14, 2020 at 15:41

You must log in to answer this question.

Not the answer you're looking for? Browse other questions tagged .